100 POINTS!!!

Question


100 POINTS!!!Question

Answers

Answer 1

Step-by-step explanation:

A.

Sequence 2:

25-2×5=15

25-2×6=13

25-2×7=11

25-2×8=9

25-2×9=7

Ordered pairs :

(13,15)

(16,13)

(19,11)

(21,9)

(24,7)

B.

Sequence1 :

The rule is 3x+2 for x starts by 1, so:

3×1+2=5

3×2+2=8

3×3+2=11

3×4+2=14

3×5+2=17

Sequence2

The rule is 2x+3 forx starts by 6, so :

2×6+3=15

2×7+3=17

2×8+3=19

2×9+3=21

2×10+3=23

Ordered pairs

(5,15)

(8,17)

(11,19)

(14,21)

(17,23)

Answer 2
Answer:A.Step-by-step explanation:Sequence 2:25-2×5=1525-2×6=1325-2×7=1125-2×8=925-2×9=7Ordered pairs :(13,15)(16,13)(19,11)(21,9)(24,7)B.Sequence1 :The rule is 3x+2 for x starts by 1, so:3×1+2=53×2+2=83×3+2=113×4+2=143×5+2=17Sequence2The rule is 2x+3 forx starts by 6, so :2×6+3=152×7+3=172×8+3=192×9+3=212×10+3=23Ordered pairs(5,15)(8,17)(11,19)(14,21)(17,23)


Related Questions

The hypotenuse of a certain right triangle is twice the length of the
shorter leg. The shorter leg is 4 cm.

Answers

Answer:

Step-by-step explanation:

If the hypotenuse of a right triangle is twice the length of the shorter leg and the shorter leg is 4 cm, then the hypotenuse is 2 * 4 cm = 8 cm.

Using the Pythagorean theorem, we can find the length of the longer leg. The Pythagorean theorem states that in a right triangle, the square of the length of the hypotenuse is equal to the sum of the squares of the lengths of the other two sides. Letting c represent the length of the hypotenuse and a and b represent the lengths of the other two sides, we can write this as c^2 = a^2 + b^2.

Substituting in the known values for c and one of the other sides (let’s say a), we have:

8^2 = 4^2 + b^2 64 = 16 + b^2 b^2 = 48 b = sqrt(48)

So, the length of the longer leg is sqrt(48) cm, or approximately 6.93 cm.

PLEASE HELP I have 30 minutes! What is the end behavior of this equation and how did you get it?

Answers

The limit of the function will be:

lim f(x) = ∞ as x → ∞

lim f(x) = 0 as x → -∞

How to explain the function

The limit of f(x) as x goes towards infinity is infinite while the limit of f(x) approaches 0 when x tends to a negative value. Mathematically speaking,

lim f(x) = ∞ as x → ∞

lim f(x) = 0 as x → -∞

We can comprehend such behavior because the function's base is greater than 1 which explains how its growth accelerates as x increases and diminishes quickly as x declines towards zero.

Leans more about functions on

https://brainly.com/question/11624077

#SPJ1

Choose yes or no to tell whether the equation has the given solution 1/4×+3=3=4×+ 1,x=8

Answers

Answer:

[tex]\large\boxed{\tt No.}[/tex]

Step-by-step explanation:

[tex]\textsf{We are asked to determine if the equation has a solution given a value.}[/tex]

[tex]\textsf{Since x is given to us, we can use the \underline{Substitution Property of Equality} to evaluate}[/tex]

[tex]\textsf{the equation.}[/tex]

[tex]\large\underline{\textsf{What is the Substitution Property of Equality?}}[/tex]

[tex]\textsf{The Substitution Property of Equality is a Property that allows us to substitute}[/tex]

[tex]\textsf{a known term, or expression for an unknown variable, or some kind of placeholder.}[/tex]

[tex]\textsf{After Substitution, we can prove that the expressions are still equal with this}[/tex]

[tex]\textsf{property.}[/tex]

[tex]\large\underline{\textsf{Solving;}}[/tex]

[tex]\tt \frac{1}{4} x + 3 = " 3 = " 4x+1[/tex]

[tex]\textsf{There's likely a typo in your equation given, hence I will fix it.}[/tex]

[tex]\tt \frac{1}{4} x + 3 = 4x+1[/tex]

[tex]\textsf{Substitute 8 for x in both sides of the equation using our property.}[/tex]

[tex]\underline{\textsf{Use the Substitution Property of Equality;}}[/tex]

[tex]\tt \frac{1}{4} (8) + 3 = 4(8)+1[/tex]

[tex]\underline{\textsf{Follow PEMDAS, multiply first;}}[/tex]

[tex]\tt 2 + 3 = 32+1[/tex]

[tex]\underline{\textsf{Evaluate;}}[/tex]

[tex]\tt 5 \neq 33[/tex]

[tex]\textsf{If the "typo" was the actual equation;}[/tex]

[tex]\tt 5 \neq 3 \neq 33[/tex]

[tex]\large\boxed{\tt No.}[/tex]

A number rounded to the nearest thousand is 47,000 which number could be the number that was rounded

Answers

Answer:

Anything greater than or equal to 46,500, and less than or equal to 47,499 could be the answer.

For example, a number, let's say 46,589 falls within that range and may have been the number that was rounded.

cot0 equals 6, lies in quadrant 3 sin20

Answers

The exact value of sin 2θ is 12/37

How to find the exact value of sin 2θ?

Trigonometry deals with the relationship between the ratios of the sides of a right-angled triangle with its angles.

We have:

cot θ = 6

Thus, tan θ = 1/6

Using the given information, we can sketch the location of the angle θ in the quadrant (See the attached image).

Thus, we can calculate the value of the hypotenuse using the Pythagoras theorem. That is:

hypotenuse = √((-6)² + (-1)²) = √37

sin θ = -1/√37

cos θ = -6/√37

Using trig. identity:

sin 2θ = 2sinθ·cosθ

sin 2θ = 2 * (-1/√37) * (-6/√37)

sin 2θ = 12/37

Therefore, the exact value of sin 2θ is 12/37

Learn more about Trigonometry on:

brainly.com/question/11967894

#SPJ1

Complete Question

If cot θ = 6,and θ lies in quadrant 3, find the exact value of sin 2θ

Ted made $56 in interest by placing $700 in a savings account with simple interest for 1 year. What was the interest rate?

Answers

The interest rate if Ted made $56 in interest for 1 year on a principle of $700 will be 8%.

Given, the interest amount is $56.

The principal amount is $700.

The time period of investment is 1 year.

Now, we have to find the interest rate.

The formula to find the simple interest is,

  S.I = PTR/100.

Now, we need to substitute the values in the above formula.

S.I = PTR/100

56 = 700×1×R / 100

56 = 700R/100

5600 = 700R

R = 5600/700

R = 8

Therefore, the interest rate is 8%.

To know more about Simple Interest, click on:https://brainly.com/question/25845758

For the right triangle below, find the measure of the angle.
Figure is not to scale.

Answers

For the given right-angled triangle the measure of the angle is 11.30°.

Given adjacent base of the triangle =  10

given opposite side of the triangle = 2

To find out the angle we can use a little bit of trigonometry. By trigonometry, we know that tan ∠ = opposite side / adjacent side.

So, we can use the above tan ∠ relation to find the angle.

Tan ∠ = opposite side / adjacent side

Tan ∠ = 2/10

Tan ∠ = 1/5

∠ = Tan⁻¹(1/5)

∠ = 11.30°.

From the above explanation, we can conclude that the measure of the angle for the given right triangle is 11.30°.

To know more about angles,

https://brainly.com/question/25716982\

#SPJ1

Correct answer gets brainliest!!!!

Answers

It is a point and point is zero dimensional. Therefore the right options are A and C respectively.

Understanding Point in Mathematics

Point is a basic geometric object that represents a location or a position in space. It is often represented as a dot or a small circle.

Points have no size, shape, or dimension. They are considered to be zero-dimensional objects, and they are often used as a building block for more complex geometric structures.

In addition to representing locations in space, points are also used in other areas of mathematics, such as in coordinate systems and in topology. In geometry, points are used to define lines, planes, and other geometric shapes.

Learn more about point here:

https://brainly.com/question/28551043

#SPJ1


If mZRST = 70, mZQST = 2x, and mZQSR = 3x - 10, what is the mZQSR
16
48
32
38

Answers

Thus, the value of x for the given set of adjacent angles is found as: x = 16 and m∠QSR = 38.

Explain about the adjacent angles:

If two angles share a side and a vertex, they are said to be neighbouring in geometry. In other words, neighbouring angles do not overlap and are placed next to one another immediately.

We may infer from our criteria and the aforementioned instances that any pair of neighbouring angles has a shared vertex and a common side. They really aren't adjacent if one of these elements is absent. By searching for these two characteristics, we can categorise pairs of angles as neighbouring or not adjacent.There are numerous unique connections between angles in pairs. You can recognise other angle connections, such as supplementary and complementary angles, by recognising nearby angles.

Given data:

m∠RST = 70, m∠QST = 2x, and m∠QSR = 3x - 10

From the figure:

m∠RST  = m∠QST + m∠QSR

Put the values

70 = 2x + 3x - 10

5x = 80

x = 16

Thus,

m∠QSR = 3x - 10  = 3(16) - 10

m∠QSR = 38

Thus, the value of x for the given set of adjacent angles is found as: x = 16 and m∠QSR = 38.

Know more about the adjacent angles:

https://brainly.com/question/28394984

#SPJ1

Complete question:

If m∠RST = 70, m∠QST = 2x, and m∠QSR = 3x - 10, what is the m∠QSR?.

The figure is attached:

16

48

32

38

Create a bar graph from the results of the following survey question: "On average, how much time do you spend using your cell phone each week?"

Answers

The bar graph from the results of the following survey question titled "Hours spent using your cell phone each week" is found in the attachment.

What is a bar graph?

A bar chart, also called a bar graph, is a visual representation of categorical data that uses rectangular bars with heights or lengths that correspond to the measure of the values they represent.

The bars in a bar chart can be plotted either vertically or horizontally.

Vertical bars, horizontal bars, grouped bars, or stacked bars can all be used to make bar graphs.

Learn more about bar graphs at: https://brainly.com/question/30243333

#SPJ1

For the function f(x)=|x-3 | select all the true statements. A. The function is increasing on the interval [3, 5) B. The function is increasing of the Interval (0, 3) C. The vertex is (0, 3) D. The vertex is (3, 0). E. The y-intercept is (0, 3). F. The y-intercept is (3, 0).​

Answers

Answer:

D and E

----------------------

See attached graph to help with answer choices.

Given function:

f(x) = | x - 3 |

Answer choices:

A. The function is increasing on the interval [3, 5) - False, correct interval is [3, + ∞)B. The function is increasing on the Interval (0, 3) - False, as shown aboveC. The vertex is (0, 3) - False, the vertex is (3, 0)D. The vertex is (3, 0) - TRUEE. The y-intercept is (0, 3) - TRUEF. The y-intercept is (3, 0) - False, the correct one is given above

Identify an equation in standard form for ellipse with its center at the origin, a vertex at (3, 0), and a focus at (1, 0). HELP ASAP! This is due in 15 minutes!!

Answers

Answer:

Step-by-step explanation:

The standard form equation for an ellipse with center at the origin is:

x^2/a^2 + y^2/b^2 = 1

where 'a' is the distance from the center to the vertices along the x-axis and 'b' is the distance from the center to the vertices along the y-axis.

In this case, the center is at the origin, and a vertex is at (3, 0). So, we know that 'a' = 3.

We also know that the distance from the center to the focus is 'c', and that:

c^2 = a^2 - b^2

Since the center is at the origin, 'c' is the distance from the focus (1, 0) to the origin, which is 1. So, we can solve for 'b' as:

c^2 = a^2 - b^2

1^2 = 3^2 - b^2

b^2 = 3^2 - 1^2

b^2 = 8

b = sqrt(8) = 2sqrt(2)

Substituting these values into the standard form equation, we get:

x^2/3^2 + y^2/(2sqrt(2))^2 = 1

Simplifying:

x^2/9 + y^2/8 = 1

So, the equation in standard form for the given ellipse is:

9x^2 + 8y^2 = 72

You’re planning to buy a car and you want to have $25,000 to purchase a car in cash in 5 years. If you’re able to receive 10% interest rate for investing the money, how much money should you invest today in order to have $25,000 in 5 years?

Answers

You have to invest $15,506.08

How to solve for the investment

PV = FV / (1 + r)^n

where:

PV = present value (amount of money you need to invest today)

FV = future value (amount of money you want to have in 5 years, which is $25,000)

r = interest rate (10% or 0.1)

n = number of periods (5 years)

Plugging in the values into the formula:

PV = $25,000 / (1 + 0.1)^5

PV = $25,000 / 1.61051

PV = $15,506.08

Therefore, you need to invest approximately $15,506.08 today at a 10% interest rate in order to have $25,000 in 5 years to purchase a car in cash.

Read more on investments here:https://brainly.com/question/27717275

#SPJ1

Hey folks help me out for some points :)

Answers

Answer:

C

Step-by-step explanation:

Positive numbers represent a positive situation. And negative numbers coincide with a negative situation.

 Ex: An increase in temperature corresponds to a positive number, and a decrease in temperature with a negative number.

Is 93 + 7 positive or negative?

Answers

Answer:

93 + 7 = 100

Step-by-step explanation:

I would assume positive because 100 is above the x-axis not below it.

Construct a truth table for the statement (~ pVq) →q.

Answers

Here is the truth table for the statement (~ p V q) → q:

```

p     q     ~p    ~p V q   (~p V q) → q

---------------------------------------

T     T     F      T           T

T     F     F      F           T

F     T     T      T           T

F     F     T      F           T

```

In the table, `p` and `q` represent the truth values of the propositions `p` and `q`, respectively. The symbol `~` represents negation (i.e., "not"). The symbol `V` represents the logical connective "or" (i.e., "inclusive or"). The symbol `→` represents the conditional connective "implies" (i.e., "if...then").

To fill in the truth table, we first evaluate `~p` and `~p V q` for each combination of truth values for `p` and `q`. Then, we evaluate `(~p V q) → q` for each combination of truth values.

We can see that the statement is always true, regardless of the truth values of `p` and `q`, except for the case where `p` is true and `q` is false.

HELP ASAP!!!
(Appropriate Measures MC)

A charity needs to report its typical donations received. The following is a list of the donations from one week. A histogram is provided to display the data.

10, 10, 10, 10, 15,15,15,19, 20, 20, 20, 25, 25, 25, 30, 30, 55, 55

A graph titled Donations to Charity in Dollars. The x-axis is labeled 10 to 19, 20 to 29, 30 to 39, 40 to 49, and 50 to 59. The y-axis is labeled Frequency. There is a shaded bar up to 8 above 10 to 19, up to 6 above 20 to 29, up to 2 above 30 to 39, and up to 2 above 50 to 59. There is no shaded bar above 40 to 49.

Which measure of variability should the charity use to accurately represent the data? Explain your answer.

The IQR of 10 is the most accurate to use, since the data is skewed.
The range of 10 is the most accurate to use, since the data is skewed.
The IQR of 45 is the most accurate to use to show that they need more money.
The range of 45 is the most accurate to use to show that they have plenty of money.

Answers

A measure of variability which the charity should use to accurately represent the data include the following: D. The range of 45 is the most accurate to use to show that they have plenty of money.

What is a box-and-whisker plot?

In Mathematics and Statistics, a box plot is a type of chart that can be used to graphically or visually represent the five-number summary of a data set with respect to locality, skewness, and spread.

Based on the information provided about the data set, the five-number summary for the given data set include the following:

Minimum (Min) = 10.

First quartile (Q₁) = 13.75.

Median (Med) = 20.

Third quartile (Q₃) = 26.25.

Maximum (Max) = 55.

For the IQR, we have:

IQR = Third quartile (Q₃) - First quartile (Q₁)

IQR = 26.25 - 13.75

IQR = 12.50

For the range, we have;

Range = maximum - minimum

Range = 55 - 10

Range = 45.

Read more on range here: https://brainly.com/question/31352873

#SPJ1

The ratio of two numbers is 7 to 3 and the sum of their squares is 232. Find the numbers.

Answers

Let the two numbers be 7x and 3x.

We know that (7x)^2 + (3x)^2 = 232.

Expanding this equation, we get 58x^2 = 232.

Dividing both sides by 58, we get x^2 = 4.

Therefore, x = +/- 2.

Plugging this value of x into the expression for the two numbers, we get that the numbers are 14 and 6, or -14 and -6.
Let's call the two numbers in this problem "x" and "y". We know that the ratio of x to y is 7 to 3, so we can write:

x/y = 7/3

We can use this information to solve for one of the variables in terms of the other. For example, we can solve for y in terms of x:

y = (3/7) x

Now we know that one of the numbers is equal to (3/7) times the other. We also know that the sum of their squares is 232, so we can write:

x^2 + y^2 = 232

Substituting y = (3/7) x, we get:

x^2 + (3/7)^2 x^2 = 232

Simplifying, we get:

(1 + (3/7)^2) x^2 = 232

(1 + 9/49) x^2 = 232

(58/49) x^2 = 232

x^2 = (232 * 49) / 58

x^2 = 196

Taking the square root of both sides, we get:

x = 14

Now we can use the equation y = (3/7) x to find y:

y = (3/7) * 14 = 6

Therefore, the two numbers are 14 and 6.

1a. Model the periodic motion of the horses using an
equation and a graph.
Horse Equation: Use function notation in degrees. Your
function will graph below

Answers

The amount of time it takes the horse to complete a full cycle of motion is known as the period of the motion and can be calculated as follows:

T = 2π/ω.

What is the explanation for the above response?

Let's assume that a straightforward harmonic motion can be used to simulate the horse's movements. The simple harmonic motion equation is:

y = A sin(ωt + φ)

where:

The deviation from equilibrium is represented by the number y.

The maximum displacement's amplitude is A.

The angular frequency is equal to two times the frequency ω.

It's time t

The phase angle is φ

The equation can be written as follows, assuming that the horse is moving vertically and that its highest point corresponds to y = 0.

y = A sin(ωt)

where is the angular frequency and A is the motion's amplitude.

On a graph, where the horizontal axis denotes time and the vertical axis denotes displacement, we can draw this function as a sine wave. The

amount of time it takes the horse to complete a full cycle of motion is known as the period of the motion and can be calculated as follows:

T = 2π/ω

We need to know the duration of the horse's motion in order to predict when Ivan will have another chance to make a flawless shot. It's challenging to determine the period precisely in the absence of more details.

You can name the graph of the carousel equation θ = A cos(ωt) as follows:

The time, t, is displayed on the x-axis as seconds.

The angular position from the midline, θ, expressed in degrees, is shown on the y-axis.

The horizontal axis at θ = 0 degrees is the midline.

The largest angle of departure from the midline, or the distance between the wave's crest (highest point) or trough (lowest point), is known as the amplitude, or A.

The interval between two consecutive wave crests or troughs is known as the period, or T, which is the length of time it takes for one full cycle of motion.

To know more about angular position, visit:

brainly.com/question/2005412

#SPJ1

How many games did the team play last season?

Answers

The number of games that the team did play in the last season is 23.

What is the frequency?

The frequency is the number of times an event happens. Therefore, frequency is the number of repetitions of a digit or an event.

The given frequency table represents the frequency of different runs scored by the team in the entire season.

The frequency is written in tally form, which means that each of the vertical lines represents the count of 1, while a group of fours lines such that the four lines are vertical while the fifth line cuts the other four represents a count of 5 as shown in the second column of the second row.

Therefore, the frequency table can be made as:

Number of runs     Frequency

            0                         8

            1                          5

            2                         3

            3                         6

            4                         1

Total:                              23

Hence, the team played 23 games.

Learn more about Frequency:

https://brainly.com/question/5102661

#SPJ1

The table below shows the number of people in an arena t minutes after an event has ended.
T= minutes after event ended: 4 8 12 16 20 24
F(t)= number of people in area: 15221 10649 7375 4859 3468 2285
-Use the regression capability of your graphing calculator to find an exponential equation matching this data. Round values to three decimal places.
F(T)= ___
-use your equation (with values rounded to three decimal places) to help you answer the following questions.
(Note: do not use the data table to answer these questions) Round your answers to the nearest whole number.
- How many people were in the arena 23 minutes after the event ended?
-How many people were in the arena 60 minutes after the event ended?
How many people were in the arena at the exact moment the event ended?

Answers

Approximately 19142 people were in the arena at the exact moment the event ended.

How to solve

To decipher an exponential equation synchronous with the provided data, we will employ the equation of [tex]F(t) = a * b^t.[/tex]

Herein, 'F(t)' stands for the total number of attendees in the stadium at 't' minutes after the stoppage of the event, with 'a' symbolizing the initial value and 'b' being the base.

Exploiting the regression aptitudes of a graphing calculator, the exponential equation that most adequately applies to the listed information is: [tex]F(t) = a * b^t[/tex] with three decimal points specified.

F(t) = 19141.846 * 0.718^t

Using the equation:

[tex]F(23) = 19141.846 * 0.718^2^3 = 2726[/tex]

Answer: Approximately 2726 people were in the arena 23 minutes after the event ended.

How many people were in the arena 60 minutes after the event ended?

[tex]F(60) = 19141.846 * 0.718^6^0 = 123[/tex]

Answer: Around 123 people were in the arena a full hour after the occurrence concluded.

How many people were present precisely when the event wrapped up?

F(0) = 19141.846 * 0.718^0 = 19141.846 * 1 = 19141.846

Answer: Approximately 19142 people were in the arena at the exact moment the event ended.

Read more about exponential function here:

https://brainly.com/question/2456547

#SPJ1

find the probability that a randomly selected point within the large square falls in the red shaded square 6 6 15 15 p= enter as a decimal rounded to the nearest hundredth

Answers

The probability that a randomly selected point within the red-shaded square is 0.16

Finding the probability of a point

From the question, we have the following parameters that can be used in our computation:

Red square of 6 by 6White square of length 15

The areas of the above shapes are

Red square = 6 * 6 = 36

White square = 15^2 = 225

The probability is then calculated as

P = Red square /White square

So, we have

P = 36/225

Evaluate

P = 0.16

Hence, the calculated value of the probability that the point falls in the red square is 0.16

Read more about probability at

brainly.com/question/251701

#SPJ1

Solve 8 • w ≤ 72. Graph the solution.
Please help

Answers

Answer:

w≤9

Step-by-step explanation:

8w ≤ 72

Divide by 8 on both sides

w ≤ 9

For graph, draw a number line and label a point as 9. Draw a solid line facing to the left and having a closed point at 9. The line should not extend past 9.

M7|L5
Does this shape have at least 1 pair of parallel lines? 40
Has at least 1 pair of parallel lines
Does not have at least 1 pair of parallel
lines
More
Enter ✔

Answers

Answer:

Does this shape have at least 1 pair of parallel lines? 40

Has at least 1 pair of parallel lines

Does not have at least 1 pair of parallel

lines

More

What is the surface area of this composite solid?

Answers

The surface area of the given composite solid is calculated as: 200.24 square units.

How to Find the Surface Area of the Composite Solid?

Surface area of the composite solid = surface area of rectangular prism + surface area of cylinder - 2(area of the base of the cylinder)

Surface area of rectangular prism = length * width * height = 10 * 3 * 5

= 150 square units.

Surface area of cylinder = 2πr(h + r)

height (h) = 4

Radius (r) = 2

Plug in the values:

Surface area of cylinder = 2 * 3.14 * 2(4 + 2) = 75.36 square units

Area of the base of the cylinder = πr² = 3.14 * 2²

= 12.56 square units

Thus, we have:

Surface area of the composite solid = 150 + 75.36 - 2(12.56) = 200.24 square units.

Learn more about Surface area line on:

https://brainly.com/question/28178861

#SPJ1

Sarah uses a square and two semicircular7 inches regions to design a heart shaped poster find the area of the heart then find the approximate area by using 3.14radius

Answers

The poster has an area equal to 192.423 square inches.

How to determine the area of a heart shaped poster

In this problem we must determine the area of a heart shaped poster, that is, the combination of the areas of a square and two semicircles, whose resulting formula is introduced below:

A = D² + (π / 4) · D²

Where:

A - Area of the heart shaped poster, in square inches.D - Side length of the square, in inches.

If we know that π = 3.14 and D = 7 in, then the area of the poster is:

A = 7² + (π / 4) · 7²

A = (5π / 4) · 7²

A = 192.423 in²

The area of the poster is equal to 192.423 square inches.

Remark

The image is missing and the statement is not well explained. A correct statement is shown below:

Sarah uses a square with side length of 7 inches and two semicircles with diameter of 7 inches to design a heart shaped poster. Find the area of the heart. Then, find the approximate area by using 3.14.

To learn more on areas of composite figures: https://brainly.com/question/30488703

#SPJ1

1. Ashenge Private Limited Company is about to start production of new products. Before the start of the new production, it cost the company $200,000 to construct the factory building. Once the construction is over, the cost per unit of production is estimated to be Birr 20. Upon selling, the company will incur a 20% commission expense. Moreover, the price per unit of products is decided to be Birr 50.
Required:
a) Construct the total cost equation in terms of quantity.
b) Determine the break-even quantity and break-even revenue.

Answers

The total cost equation is 200,000 + 40n and the break-even revenue is 20,000.

Given that a company is starting a new production, it cost the company $200,000 to construct the factory building.

The cost per unit of production is estimated to be Birr 20.

The company will incur a 20% commission expense.

The price per unit of products is decided to be Birr 50.

We need to find the total cost equation in terms of quantity and determine the break-even quantity.

a) The fixed cost of the company is $200,000, and 40 per unit sell,

Therefore for n units the total cost =

200,000 + 40n

b) BEQ = FC / P−VC

= 200,000 / 50-40

= 20,000

Hence the total cost equation is 200,000 + 40n and the break-even revenue is 20,000.

Learn more about break-even revenue click;

https://brainly.com/question/28504810

#SPJ1

Husk takes out a 3200 student loan at 6.7% to help him

Part (a) find the total amount interest that will accrue for loan 1(community college )

Part (b) find the total amount of intrest that will a cure for loan 2 state university
Part (c) find the total amount intrest that accrues until payment begin

Answers

(c) Total interest accrued until payments begin = $1023.33 (loan 1) + $2362.33 (loan 2) ≈ $3385.66.

How to solve

(a) For loan 1, $3200 at 6.7% for 4 years and 7 months (4 years in college + 3 months deferment), the simple interest = 3200 * 0.067 * (55/12) ≈ $1023.33.

(b) For loan 2, $12,000 at 7.3% for 2 years and 7 months (2 years in college + 3 months deferment), simple interest = 12000 * 0.073 * (31/12) ≈ $2362.33.

(c) Total interest accrued until payments begin = $1023.33 (loan 1) + $2362.33 (loan 2) ≈ $3385.66.

Read more about interest rates here:

https://brainly.com/question/25793394

#SPJ1

Someone help i really need help with this

Answers

The equation which can be used to find the amount the admission tickets cost would be 4x + 25 = 175. The cost of the admission tickets would therefore be $ 37.50.

How to find the equation ?

The total spent by Ms. Boi was $ 175.00 so this is what the entire equation would be equal to. The cost of admission tickets will be denoted as x because they are the unknown figure.

There are 4 tickets purchased so the equation would be:

4 x Cost of tickets - Parking cost = Total spent

4 x - 25 = 175

The cost of each admission ticket is:

4 x - 25 = 175

x = 150 / 4

x = $ 37.50

Find out more on cost at cost

#SPJ1

If a, b, c are in H.P.; prove that b+ca+ca+b ac' ab bc are also in A.P.​

Answers

If a, b, c are in Harmonic Progression, b+ca+ca+b ac' ab bc are in Arithmetic Progression.

How to prove Harmonic Progression and Arithmetic Progression?

Given a, b, c are in H.P.:

1/a + 1/b = 2/c

Multiplying both sides by abc:

bc + ac = 2ab

Adding c to both sides:

bc + ac + c = 2ab + c

Rearranging the terms:

c + ab = ac + bc

Adding the terms b + ca and ca + b to both sides:

b + ca + ca + b + ac' + ab + bc = ac + bc + b + ca + ca + b

Simplifying:

b + ca + ca + b + ac' + ab + bc = 2ac + 2b

Dividing both sides by 2:

(b + ca + ca + b + ac' + ab + bc)/2 = ac + b

Hence, b+ca+ca+b ac' ab bc are in A.P.

Find out more on H.P here: https://brainly.com/question/27927304

#SPJ1

Other Questions
In June, Christy Sports has to determine how many Obermeyer jackets to order for the ski season that will start late fall. Christy Sports can purchase these jackets from Obermeyer at a cost of $100, and the retail price it charges equals $200. Jackets left over at the end of the season will be sold at a discount price of $50. Christy Sports has to order jackets in multiples of 25. Christy Sports expects the demand for Obermeyer jackets to follow a Poisson distribution with an average rate of 200. a. Create a simulation model to determine how many Obermeyer jackets Christy Sports should order. What is the optimal order quantity? b. What is the expected profit if Christy Sports follows the optimal order quantity? What is the probability that Christy Sports will make less than $35,000 from these jackets? Suppose that 42% of students of a high school play video games at least once a month. The computerprogramming club takes an SRS of 30 students from the population of 792 students at the school and finds that40% of students sampled play video games at least once a month. The club plans to take more samples like this. Let represent the proportion of a sample of 30 students who play video games at least once a month. What are the mean and standard deviation of the sampling distribution of p?Choose 1 answer:Hy = 0. 42Op =0. 42 (0. 58)30Hg = (30)(0. 42))Op = 130(0. 42)(0. 58) A limited partner will not lose his limited status because of voting on an amendment to the partnership agreement allowing a new business venture. True. False. Queen noor of jordan is a leader of which form of government?. mr. green uses nsaids regularly to control chronic pain and complains of frequent stomach pain. the nurse recognizes this as gastritis and realizes that he may not be at risk for deficiencies of what happens to a rock when its weathered? A It is moved by wind, air, or water Help with the problem in photo please! Select the correct answer.consider functions fandi-408-2432g(x)1i2-23-44-8what is the value of x when (fog)(x) = -8? Consider the following. g(x) = 8ex; h(x) = 8(6.5) (a) Write the product function. = f(x) = (b) Write the rate-of-change function. f'(x) = Consider the following. g(x) = 9e- + In(x); h(x) = 4x(a) Write the product function. f(x) = (b) Write the rate-of-change function. f'(x) = Suppose you invest $5000 in a savings account that pays an apr of 4%. If the interest is compounded monthly, what is the balance in the account after 10 years?. anyone know how to answer this? According to each of the Utilitarian and Capabilities approacheswhat reasons should motivate an engineer to attend to the needs of the visually impaired? eli whitney's invention of the cotton gin changed the industry of the south in which way?group of answer choicesit created greater competition between southern cotton farmers.it led to greater industrialization in the south.it led to more land in the south being dedicated to cotton farming.it decreased the dependence on slave labor in cotton farming. m company's last dividend was $1.25 (d0). the dividend growth rate is expected to be constant at 15.0% for 2 years, after which dividends are expected to grow at a rate of 6% forever. if the firm's required return (rs) is 11%, what is its d3 (dividend at the end of year 3) ? A. My parents to New York. (get) married before they (move) b. Harry. Jane. (work) as a driver when he (meet) c. Josh with Amy. (be) very sad because he (split up) d. By the time we (arrive) the film already e. You obviously. (start). (not listening) when I (tell) you to close the window. F. As soon as Clara g. 1. (pass) her exams, she (rent) a flat. (look out) of the window and (stop) raining. (notice) that it (go) straight to bed. (rain), so we j. It walk. (decide) not to go for a k. My mum games. \ (call)me while I (play) computer 1. 1 (not know) where you were, because you (not call) me. M. Peter (have) his mobile for three years when it (break down) n. Simon and Mary (know) each other for five years before they (get) married last month. H. What i. When I you (come) home I (do) when I (call) you? (have) dinner and then What needs are more necessary Maslow tan others ? What evidence is provided to explain this priority? Erica spins a spinner with equal sections numbered 1 through 4 and selects a colored tile from a bag. Based on the tree diagram given, what is the probability of spinning a 2 or 3 on the spinner and drawing a blue tile? Who are the three people that went to Gatsbys funeral?1. _____________2. _____________3. _______________No cheating. It will not be tolerated Find (Round your answer to the nearest hundredth) A rocket in outer space is traveling toward a far-off planet. An astronaut turns on the rockets engines, which exerts a force on the rocket. There is no gravity or air resistance. While the engines are on, how will the rocket move?